Pointwise convergence of average of continuous functions












3












$begingroup$


Suppose that $(f^n)$ is a sequence of continuous functions from a compact metric space $A$ into a convex compact subset of the reals $B$. Is there a subsequence, $(f^{n_k})$, such that the sequence



$f^{n_1},(1/2)f^{n_1}+(1/2)f^{n_2},(1/3)f^{n_1}+(1/3)f^{n_2}+(1/3)f^{n_3},...$



converges pointwise to some map $f:Ato B$?










share|cite|improve this question











$endgroup$

















    3












    $begingroup$


    Suppose that $(f^n)$ is a sequence of continuous functions from a compact metric space $A$ into a convex compact subset of the reals $B$. Is there a subsequence, $(f^{n_k})$, such that the sequence



    $f^{n_1},(1/2)f^{n_1}+(1/2)f^{n_2},(1/3)f^{n_1}+(1/3)f^{n_2}+(1/3)f^{n_3},...$



    converges pointwise to some map $f:Ato B$?










    share|cite|improve this question











    $endgroup$















      3












      3








      3





      $begingroup$


      Suppose that $(f^n)$ is a sequence of continuous functions from a compact metric space $A$ into a convex compact subset of the reals $B$. Is there a subsequence, $(f^{n_k})$, such that the sequence



      $f^{n_1},(1/2)f^{n_1}+(1/2)f^{n_2},(1/3)f^{n_1}+(1/3)f^{n_2}+(1/3)f^{n_3},...$



      converges pointwise to some map $f:Ato B$?










      share|cite|improve this question











      $endgroup$




      Suppose that $(f^n)$ is a sequence of continuous functions from a compact metric space $A$ into a convex compact subset of the reals $B$. Is there a subsequence, $(f^{n_k})$, such that the sequence



      $f^{n_1},(1/2)f^{n_1}+(1/2)f^{n_2},(1/3)f^{n_1}+(1/3)f^{n_2}+(1/3)f^{n_3},...$



      converges pointwise to some map $f:Ato B$?







      functional-analysis metric-spaces compactness uniform-convergence pointwise-convergence






      share|cite|improve this question















      share|cite|improve this question













      share|cite|improve this question




      share|cite|improve this question








      edited Dec 7 '18 at 20:44









      Alex Ravsky

      40.4k32282




      40.4k32282










      asked Oct 22 '18 at 12:45









      mo15mo15

      1768




      1768






















          1 Answer
          1






          active

          oldest

          votes


















          1












          $begingroup$

          The answer depends on the sequence $(f^n)$.



          Since the sequence $(f^n)$ is uniformly bounded, if it is equicontinuous then by Arzelà–Ascoli theorem for compact Hausdorff spaces, it is relatively compact in the space $C(A)$ of real-valued continuous functions on $A$. So it contains a subsequence $(f^{n_i})$, uniformly convergent to a function $fin C(A)$. Then a sequence $left(frac 1ksum_{i=1}^k f^{n_i}right)$ uniformly converges to $f$ too.



          But for general case the claim may fail. Consider the following example. Let $A$ be the Cantor cube, that is a countable power of a discrete space ${0,1}$ endowed with the Tychonoff product topology. For each $n$ let $f^n:Ato [0,1]$ be the projection at the $n$-th coordinate. Let $(n_i)$ be an arbitrary increasing sequence of natural numbers. It is easy to construct a set $DsubsetBbb N$ with undefined density, that is such that the limit $d(D)=lim_{ktoinfty} frac 1k|{ ain D: ale k}|$ does not exist. Pick any point $x=(x_n)in A$ such that for each natural $k$ we have $x_{n_k}=1$, if $kin D$ and $x_{n_k}=0$, otherwise. Since for each $k$ we have $|{ ain D: ale k}|=sum_{i=1}^k f^{n_i}(x)$, the limit $frac 1ksum_{i=1}^k f^{n_i}(x)$ does not exist.






          share|cite|improve this answer











          $endgroup$













          • $begingroup$
            Thanks for your answer. I am little confused with the construction. Do you mean a set $Dsubseteq{0,1}^N$ such that $lim_{ktoinfty}frac{1}{k}(d_1+dots+d_k)$ does not exist? And then pick $xin A$ such that $x_{n_k}=d_k$ for each $k$?
            $endgroup$
            – mo15
            Dec 8 '18 at 22:02










          • $begingroup$
            @mo15 Oops, I sometimes denoted the set $D$ as $A$. Sorry. Corrected.
            $endgroup$
            – Alex Ravsky
            Dec 8 '18 at 23:03










          • $begingroup$
            But $D$ must be a subset of $A$, not $N$, correct? An the limit $d(D)$ must be defined differently, I think.
            $endgroup$
            – mo15
            Dec 8 '18 at 23:15












          • $begingroup$
            In view of this example, a natural question would be whether the claim is true when $A$ is a compact subset of Euclidean space $mathbb R^n$.
            $endgroup$
            – mo15
            Dec 9 '18 at 0:42












          • $begingroup$
            No, $D$ is a subset of $Bbb N$.
            $endgroup$
            – Alex Ravsky
            Dec 9 '18 at 3:48











          Your Answer





          StackExchange.ifUsing("editor", function () {
          return StackExchange.using("mathjaxEditing", function () {
          StackExchange.MarkdownEditor.creationCallbacks.add(function (editor, postfix) {
          StackExchange.mathjaxEditing.prepareWmdForMathJax(editor, postfix, [["$", "$"], ["\\(","\\)"]]);
          });
          });
          }, "mathjax-editing");

          StackExchange.ready(function() {
          var channelOptions = {
          tags: "".split(" "),
          id: "69"
          };
          initTagRenderer("".split(" "), "".split(" "), channelOptions);

          StackExchange.using("externalEditor", function() {
          // Have to fire editor after snippets, if snippets enabled
          if (StackExchange.settings.snippets.snippetsEnabled) {
          StackExchange.using("snippets", function() {
          createEditor();
          });
          }
          else {
          createEditor();
          }
          });

          function createEditor() {
          StackExchange.prepareEditor({
          heartbeatType: 'answer',
          autoActivateHeartbeat: false,
          convertImagesToLinks: true,
          noModals: true,
          showLowRepImageUploadWarning: true,
          reputationToPostImages: 10,
          bindNavPrevention: true,
          postfix: "",
          imageUploader: {
          brandingHtml: "Powered by u003ca class="icon-imgur-white" href="https://imgur.com/"u003eu003c/au003e",
          contentPolicyHtml: "User contributions licensed under u003ca href="https://creativecommons.org/licenses/by-sa/3.0/"u003ecc by-sa 3.0 with attribution requiredu003c/au003e u003ca href="https://stackoverflow.com/legal/content-policy"u003e(content policy)u003c/au003e",
          allowUrls: true
          },
          noCode: true, onDemand: true,
          discardSelector: ".discard-answer"
          ,immediatelyShowMarkdownHelp:true
          });


          }
          });














          draft saved

          draft discarded


















          StackExchange.ready(
          function () {
          StackExchange.openid.initPostLogin('.new-post-login', 'https%3a%2f%2fmath.stackexchange.com%2fquestions%2f2965968%2fpointwise-convergence-of-average-of-continuous-functions%23new-answer', 'question_page');
          }
          );

          Post as a guest















          Required, but never shown

























          1 Answer
          1






          active

          oldest

          votes








          1 Answer
          1






          active

          oldest

          votes









          active

          oldest

          votes






          active

          oldest

          votes









          1












          $begingroup$

          The answer depends on the sequence $(f^n)$.



          Since the sequence $(f^n)$ is uniformly bounded, if it is equicontinuous then by Arzelà–Ascoli theorem for compact Hausdorff spaces, it is relatively compact in the space $C(A)$ of real-valued continuous functions on $A$. So it contains a subsequence $(f^{n_i})$, uniformly convergent to a function $fin C(A)$. Then a sequence $left(frac 1ksum_{i=1}^k f^{n_i}right)$ uniformly converges to $f$ too.



          But for general case the claim may fail. Consider the following example. Let $A$ be the Cantor cube, that is a countable power of a discrete space ${0,1}$ endowed with the Tychonoff product topology. For each $n$ let $f^n:Ato [0,1]$ be the projection at the $n$-th coordinate. Let $(n_i)$ be an arbitrary increasing sequence of natural numbers. It is easy to construct a set $DsubsetBbb N$ with undefined density, that is such that the limit $d(D)=lim_{ktoinfty} frac 1k|{ ain D: ale k}|$ does not exist. Pick any point $x=(x_n)in A$ such that for each natural $k$ we have $x_{n_k}=1$, if $kin D$ and $x_{n_k}=0$, otherwise. Since for each $k$ we have $|{ ain D: ale k}|=sum_{i=1}^k f^{n_i}(x)$, the limit $frac 1ksum_{i=1}^k f^{n_i}(x)$ does not exist.






          share|cite|improve this answer











          $endgroup$













          • $begingroup$
            Thanks for your answer. I am little confused with the construction. Do you mean a set $Dsubseteq{0,1}^N$ such that $lim_{ktoinfty}frac{1}{k}(d_1+dots+d_k)$ does not exist? And then pick $xin A$ such that $x_{n_k}=d_k$ for each $k$?
            $endgroup$
            – mo15
            Dec 8 '18 at 22:02










          • $begingroup$
            @mo15 Oops, I sometimes denoted the set $D$ as $A$. Sorry. Corrected.
            $endgroup$
            – Alex Ravsky
            Dec 8 '18 at 23:03










          • $begingroup$
            But $D$ must be a subset of $A$, not $N$, correct? An the limit $d(D)$ must be defined differently, I think.
            $endgroup$
            – mo15
            Dec 8 '18 at 23:15












          • $begingroup$
            In view of this example, a natural question would be whether the claim is true when $A$ is a compact subset of Euclidean space $mathbb R^n$.
            $endgroup$
            – mo15
            Dec 9 '18 at 0:42












          • $begingroup$
            No, $D$ is a subset of $Bbb N$.
            $endgroup$
            – Alex Ravsky
            Dec 9 '18 at 3:48
















          1












          $begingroup$

          The answer depends on the sequence $(f^n)$.



          Since the sequence $(f^n)$ is uniformly bounded, if it is equicontinuous then by Arzelà–Ascoli theorem for compact Hausdorff spaces, it is relatively compact in the space $C(A)$ of real-valued continuous functions on $A$. So it contains a subsequence $(f^{n_i})$, uniformly convergent to a function $fin C(A)$. Then a sequence $left(frac 1ksum_{i=1}^k f^{n_i}right)$ uniformly converges to $f$ too.



          But for general case the claim may fail. Consider the following example. Let $A$ be the Cantor cube, that is a countable power of a discrete space ${0,1}$ endowed with the Tychonoff product topology. For each $n$ let $f^n:Ato [0,1]$ be the projection at the $n$-th coordinate. Let $(n_i)$ be an arbitrary increasing sequence of natural numbers. It is easy to construct a set $DsubsetBbb N$ with undefined density, that is such that the limit $d(D)=lim_{ktoinfty} frac 1k|{ ain D: ale k}|$ does not exist. Pick any point $x=(x_n)in A$ such that for each natural $k$ we have $x_{n_k}=1$, if $kin D$ and $x_{n_k}=0$, otherwise. Since for each $k$ we have $|{ ain D: ale k}|=sum_{i=1}^k f^{n_i}(x)$, the limit $frac 1ksum_{i=1}^k f^{n_i}(x)$ does not exist.






          share|cite|improve this answer











          $endgroup$













          • $begingroup$
            Thanks for your answer. I am little confused with the construction. Do you mean a set $Dsubseteq{0,1}^N$ such that $lim_{ktoinfty}frac{1}{k}(d_1+dots+d_k)$ does not exist? And then pick $xin A$ such that $x_{n_k}=d_k$ for each $k$?
            $endgroup$
            – mo15
            Dec 8 '18 at 22:02










          • $begingroup$
            @mo15 Oops, I sometimes denoted the set $D$ as $A$. Sorry. Corrected.
            $endgroup$
            – Alex Ravsky
            Dec 8 '18 at 23:03










          • $begingroup$
            But $D$ must be a subset of $A$, not $N$, correct? An the limit $d(D)$ must be defined differently, I think.
            $endgroup$
            – mo15
            Dec 8 '18 at 23:15












          • $begingroup$
            In view of this example, a natural question would be whether the claim is true when $A$ is a compact subset of Euclidean space $mathbb R^n$.
            $endgroup$
            – mo15
            Dec 9 '18 at 0:42












          • $begingroup$
            No, $D$ is a subset of $Bbb N$.
            $endgroup$
            – Alex Ravsky
            Dec 9 '18 at 3:48














          1












          1








          1





          $begingroup$

          The answer depends on the sequence $(f^n)$.



          Since the sequence $(f^n)$ is uniformly bounded, if it is equicontinuous then by Arzelà–Ascoli theorem for compact Hausdorff spaces, it is relatively compact in the space $C(A)$ of real-valued continuous functions on $A$. So it contains a subsequence $(f^{n_i})$, uniformly convergent to a function $fin C(A)$. Then a sequence $left(frac 1ksum_{i=1}^k f^{n_i}right)$ uniformly converges to $f$ too.



          But for general case the claim may fail. Consider the following example. Let $A$ be the Cantor cube, that is a countable power of a discrete space ${0,1}$ endowed with the Tychonoff product topology. For each $n$ let $f^n:Ato [0,1]$ be the projection at the $n$-th coordinate. Let $(n_i)$ be an arbitrary increasing sequence of natural numbers. It is easy to construct a set $DsubsetBbb N$ with undefined density, that is such that the limit $d(D)=lim_{ktoinfty} frac 1k|{ ain D: ale k}|$ does not exist. Pick any point $x=(x_n)in A$ such that for each natural $k$ we have $x_{n_k}=1$, if $kin D$ and $x_{n_k}=0$, otherwise. Since for each $k$ we have $|{ ain D: ale k}|=sum_{i=1}^k f^{n_i}(x)$, the limit $frac 1ksum_{i=1}^k f^{n_i}(x)$ does not exist.






          share|cite|improve this answer











          $endgroup$



          The answer depends on the sequence $(f^n)$.



          Since the sequence $(f^n)$ is uniformly bounded, if it is equicontinuous then by Arzelà–Ascoli theorem for compact Hausdorff spaces, it is relatively compact in the space $C(A)$ of real-valued continuous functions on $A$. So it contains a subsequence $(f^{n_i})$, uniformly convergent to a function $fin C(A)$. Then a sequence $left(frac 1ksum_{i=1}^k f^{n_i}right)$ uniformly converges to $f$ too.



          But for general case the claim may fail. Consider the following example. Let $A$ be the Cantor cube, that is a countable power of a discrete space ${0,1}$ endowed with the Tychonoff product topology. For each $n$ let $f^n:Ato [0,1]$ be the projection at the $n$-th coordinate. Let $(n_i)$ be an arbitrary increasing sequence of natural numbers. It is easy to construct a set $DsubsetBbb N$ with undefined density, that is such that the limit $d(D)=lim_{ktoinfty} frac 1k|{ ain D: ale k}|$ does not exist. Pick any point $x=(x_n)in A$ such that for each natural $k$ we have $x_{n_k}=1$, if $kin D$ and $x_{n_k}=0$, otherwise. Since for each $k$ we have $|{ ain D: ale k}|=sum_{i=1}^k f^{n_i}(x)$, the limit $frac 1ksum_{i=1}^k f^{n_i}(x)$ does not exist.







          share|cite|improve this answer














          share|cite|improve this answer



          share|cite|improve this answer








          edited Dec 9 '18 at 3:47

























          answered Dec 7 '18 at 20:38









          Alex RavskyAlex Ravsky

          40.4k32282




          40.4k32282












          • $begingroup$
            Thanks for your answer. I am little confused with the construction. Do you mean a set $Dsubseteq{0,1}^N$ such that $lim_{ktoinfty}frac{1}{k}(d_1+dots+d_k)$ does not exist? And then pick $xin A$ such that $x_{n_k}=d_k$ for each $k$?
            $endgroup$
            – mo15
            Dec 8 '18 at 22:02










          • $begingroup$
            @mo15 Oops, I sometimes denoted the set $D$ as $A$. Sorry. Corrected.
            $endgroup$
            – Alex Ravsky
            Dec 8 '18 at 23:03










          • $begingroup$
            But $D$ must be a subset of $A$, not $N$, correct? An the limit $d(D)$ must be defined differently, I think.
            $endgroup$
            – mo15
            Dec 8 '18 at 23:15












          • $begingroup$
            In view of this example, a natural question would be whether the claim is true when $A$ is a compact subset of Euclidean space $mathbb R^n$.
            $endgroup$
            – mo15
            Dec 9 '18 at 0:42












          • $begingroup$
            No, $D$ is a subset of $Bbb N$.
            $endgroup$
            – Alex Ravsky
            Dec 9 '18 at 3:48


















          • $begingroup$
            Thanks for your answer. I am little confused with the construction. Do you mean a set $Dsubseteq{0,1}^N$ such that $lim_{ktoinfty}frac{1}{k}(d_1+dots+d_k)$ does not exist? And then pick $xin A$ such that $x_{n_k}=d_k$ for each $k$?
            $endgroup$
            – mo15
            Dec 8 '18 at 22:02










          • $begingroup$
            @mo15 Oops, I sometimes denoted the set $D$ as $A$. Sorry. Corrected.
            $endgroup$
            – Alex Ravsky
            Dec 8 '18 at 23:03










          • $begingroup$
            But $D$ must be a subset of $A$, not $N$, correct? An the limit $d(D)$ must be defined differently, I think.
            $endgroup$
            – mo15
            Dec 8 '18 at 23:15












          • $begingroup$
            In view of this example, a natural question would be whether the claim is true when $A$ is a compact subset of Euclidean space $mathbb R^n$.
            $endgroup$
            – mo15
            Dec 9 '18 at 0:42












          • $begingroup$
            No, $D$ is a subset of $Bbb N$.
            $endgroup$
            – Alex Ravsky
            Dec 9 '18 at 3:48
















          $begingroup$
          Thanks for your answer. I am little confused with the construction. Do you mean a set $Dsubseteq{0,1}^N$ such that $lim_{ktoinfty}frac{1}{k}(d_1+dots+d_k)$ does not exist? And then pick $xin A$ such that $x_{n_k}=d_k$ for each $k$?
          $endgroup$
          – mo15
          Dec 8 '18 at 22:02




          $begingroup$
          Thanks for your answer. I am little confused with the construction. Do you mean a set $Dsubseteq{0,1}^N$ such that $lim_{ktoinfty}frac{1}{k}(d_1+dots+d_k)$ does not exist? And then pick $xin A$ such that $x_{n_k}=d_k$ for each $k$?
          $endgroup$
          – mo15
          Dec 8 '18 at 22:02












          $begingroup$
          @mo15 Oops, I sometimes denoted the set $D$ as $A$. Sorry. Corrected.
          $endgroup$
          – Alex Ravsky
          Dec 8 '18 at 23:03




          $begingroup$
          @mo15 Oops, I sometimes denoted the set $D$ as $A$. Sorry. Corrected.
          $endgroup$
          – Alex Ravsky
          Dec 8 '18 at 23:03












          $begingroup$
          But $D$ must be a subset of $A$, not $N$, correct? An the limit $d(D)$ must be defined differently, I think.
          $endgroup$
          – mo15
          Dec 8 '18 at 23:15






          $begingroup$
          But $D$ must be a subset of $A$, not $N$, correct? An the limit $d(D)$ must be defined differently, I think.
          $endgroup$
          – mo15
          Dec 8 '18 at 23:15














          $begingroup$
          In view of this example, a natural question would be whether the claim is true when $A$ is a compact subset of Euclidean space $mathbb R^n$.
          $endgroup$
          – mo15
          Dec 9 '18 at 0:42






          $begingroup$
          In view of this example, a natural question would be whether the claim is true when $A$ is a compact subset of Euclidean space $mathbb R^n$.
          $endgroup$
          – mo15
          Dec 9 '18 at 0:42














          $begingroup$
          No, $D$ is a subset of $Bbb N$.
          $endgroup$
          – Alex Ravsky
          Dec 9 '18 at 3:48




          $begingroup$
          No, $D$ is a subset of $Bbb N$.
          $endgroup$
          – Alex Ravsky
          Dec 9 '18 at 3:48


















          draft saved

          draft discarded




















































          Thanks for contributing an answer to Mathematics Stack Exchange!


          • Please be sure to answer the question. Provide details and share your research!

          But avoid



          • Asking for help, clarification, or responding to other answers.

          • Making statements based on opinion; back them up with references or personal experience.


          Use MathJax to format equations. MathJax reference.


          To learn more, see our tips on writing great answers.




          draft saved


          draft discarded














          StackExchange.ready(
          function () {
          StackExchange.openid.initPostLogin('.new-post-login', 'https%3a%2f%2fmath.stackexchange.com%2fquestions%2f2965968%2fpointwise-convergence-of-average-of-continuous-functions%23new-answer', 'question_page');
          }
          );

          Post as a guest















          Required, but never shown





















































          Required, but never shown














          Required, but never shown












          Required, but never shown







          Required, but never shown

































          Required, but never shown














          Required, but never shown












          Required, but never shown







          Required, but never shown







          Popular posts from this blog

          Quarter-circle Tiles

          build a pushdown automaton that recognizes the reverse language of a given pushdown automaton?

          Mont Emei